Let's play with sums!

Algebra Level 4

If x 1 x_{1} and x 2 x_{2} are distinct roots of x 2 6 x + 1 = 0 x^{2}-6x+1=0 and S n = x 1 n + x 2 n S_{n}=x_{1}^{n}+x_{2}^{n} , where n n is a positive integer. Find the remainder of S 50 S_{50} when divided by 5.


The answer is 4.

This section requires Javascript.
You are seeing this because something didn't load right. We suggest you, (a) try refreshing the page, (b) enabling javascript if it is disabled on your browser and, finally, (c) loading the non-javascript version of this page . We're sorry about the hassle.

4 solutions

James Pohadi
Apr 16, 2017

For x = x 1 x=x_{1} and x = x 2 x=x_{2}

x 2 6 x + 1 = x 2 x + 1 = 0 (mod 5) ( x 2 x + 1 ) ( x + 1 ) = x 3 + 1 = 0 (mod 5) x 3 = 1 (mod 5) x 1 3 = x 2 3 = 1 (mod 5) \begin{aligned} x^{2}-6x+1=x^{2}-x+1&=0 \text{ (mod 5)} \\ (x^{2}-x+1)(x+1)&=x^{3}+1=0 \text{ (mod 5)} \\ x^{3}&=-1 \text{ (mod 5)} \implies x_{1}^{3}=x_{2}^{3}=-1 \text { (mod 5)}\end{aligned}

And by vieta, we have x 1 + x 2 = 6 x_{1}+x_{2}=6 and x 1 x 2 = 1 x_{1}x_{2}=1

Then, S 50 (mod 5) S_{50} \text{ (mod 5)} is S 50 = ( x 1 50 + x 2 50 ) = ( ( x 1 3 ) 16 × x 1 2 + ( x 2 3 ) 16 × x 2 2 ) (mod 5) S 50 = ( ( 1 ) 16 × x 1 2 + ( 1 ) 16 × x 2 2 ) (mod 5) S 50 = ( x 1 2 + x 2 2 ) = ( ( x 1 + x 2 ) 2 2 x 1 x 2 ) (mod 5) S 50 = ( ( 6 ) 2 2 × 1 ) (mod 5) S 50 = 34 (mod 5) S 50 = 4 (mod 5) S_{50}= (x_{1}^{50}+x_{2}^{50}) = ((x_{1}^{3})^{16} \times x_{1}^2 +(x_{2}^{3})^{16} \times x_{2}^2) \text{ (mod 5)} \\ S_{50} = ((-1)^{16} \times x_{1}^2 +(-1)^{16} \times x_{2}^2) \text{ (mod 5)} \\ S_{50} = (x_{1}^2 +x_{2}^2) =((x_{1}+x_{2})^2-2x_{1}x_{2}) \text{ (mod 5)} \\ S_{50} = ((6)^2-2 \times 1) \text{ (mod 5)} \\ S_{50} = 34 \text{ (mod 5)} \\ S_{50} = \boxed{4} \text{ (mod 5)}

Chew-Seong Cheong
Apr 15, 2017

Since x 1 x_1 and x 2 x_2 are roots of x 2 6 x + 1 = 0 x^2-6x+1=0 , by Vieta's formula we have x 1 + x 2 = 6 x_1+x_2 = 6 and x 1 x 2 = 1 x_1x_2 = 1 . And by Newton sums method, we have:

S 1 = x 1 + x 2 = 6 S 2 = x 1 2 + x 2 2 = ( x 1 + x 2 ) 2 2 x 1 x 2 = 34 S n = ( x 1 + x 2 ) S n 1 x 1 x 2 S n 2 for n 3 = 6 S n 1 S n 2 \begin{aligned} S_1 & = x_1+x_2 = 6 \\ S_2 & = x_1^2 + x_2^2 = (x_1+x_2)^2 - 2x_1x_2 = 34 \\ S_n & = (x_1+x_2)S_{n-1}-x_1x_2S_{n-2} & \small \color{#3D99F6} \text{for } n \ge 3 \\ & = 6S_{n-1} - S_{n-2} \end{aligned}

This implies that S n S_n has a recurrence relation and its characteristic equation is:

r 2 6 r + 1 = 0 r = 3 ± 8 S n = c 1 ( 3 + 8 ) n + c 2 ( 3 8 ) n where c 1 and c 2 are constants. S 1 = ( 3 + 8 ) c 1 + ( 3 8 ) c 2 S 2 = ( 3 + 8 ) 2 c 1 + ( 3 8 ) 2 c 2 = ( 17 + 6 8 ) c 1 + ( 17 6 8 ) c 2 \begin{aligned} r^2 - 6r + 1& = 0 \\ r & = 3 \pm \sqrt 8 \\ \implies S_n & = c_1 (3+\sqrt 8)^n + c_2 (3-\sqrt 8)^n & \small \color{#3D99F6} \text{where }c_1 \text{ and } c_2 \text{ are constants.} \\ S_1 & = (3+\sqrt 8)c_1 + (3-\sqrt 8)c_2 \\ S_2 & = (3+\sqrt 8)^2c_1 + (3-\sqrt 8)^2c_2 \\ & = (17+6\sqrt 8)c_1 + (17-6\sqrt 8)c_2 \end{aligned}

{ S 1 : 3 ( c 1 + c 2 ) + 8 ( c 1 c 2 ) = 6 S 2 : 17 ( c 1 + c 2 ) + 6 8 ( c 1 c 2 ) = 34 \implies \begin{cases} S_1: & 3(c_1+c_2) + \sqrt 8(c_1-c_2) = 6 \\ S_2: & 17(c_1+c_2) + 6\sqrt 8(c_1-c_2) = 34 \end{cases}

6 S 1 S 2 : c 1 + c 2 = 2 \begin{aligned} 6S_1 - S_2: \quad c_1+c_2 & = 2 \end{aligned}

S 1 : 3 ( c 1 + c 2 ) + 8 ( c 1 c 2 ) = 6 3 ( 2 ) + 8 ( c 1 c 2 ) = 6 c 1 c 2 = 0 c 1 = c 2 = 1 \begin{aligned} S_1: \quad 3(c_1+c_2) + \sqrt 8(c_1-c_2) & = 6 \\ 3(2) + \sqrt 8(c_1-c_2) & = 6 \\ c_1-c_2 & = 0 \\ \implies c_1 & = c_2 = 1 \end{aligned}

Therefore,

S n = ( 3 + 8 ) n + ( 3 8 ) n S 50 = ( 3 + 8 ) 50 + ( 3 8 ) 50 S 50 2 k = 0 25 ( 50 2 k ) 3 2 k 8 k (mod 5) Note that ( 50 2 k ) is divisible by 5 for 1 k 24 = 2 ( 3 50 + 8 25 ) (mod 5) Since gcd ( 3.5 ) = 1 , gcd ( 8 , 5 ) = 1 , Euler’s theorem applies. = 2 ( 3 50 mod ϕ ( 5 ) + 8 25 mod ϕ ( 5 ) ) (mod 5) Euler’s totient function ϕ ( 5 ) = 4 = 2 ( 3 50 mod 4 + 8 25 mod 4 ) (mod 5) = 2 ( 3 2 + 8 1 ) (mod 5) = 2 ( 17 ) (mod 5) = 4 (mod 5) \begin{aligned} S_n & = (3+\sqrt 8)^n + (3-\sqrt 8)^n \\ S_{50} & = (3+\sqrt 8)^{50} + (3-\sqrt 8)^{50} \\ S_{50} & \equiv 2 \sum_{k=0}^{25} \binom {50}{2k} 3^{2k}\cdot 8^k \text{ (mod 5)} & \small \color{#3D99F6} \text{Note that } \binom {50}{2k} \text{ is divisible by 5 for }1 \le k \le 24 \\ & = 2\left(3^{50} + 8^{25}\right) \text{ (mod 5)} & \small \color{#3D99F6} \text{Since }\gcd (3.5) =1, \gcd (8,5) =1 \text{, Euler's theorem applies.} \\ & = 2\left(3^{\color{#3D99F6} 50 \text{ mod }\phi (5)} + 8^{\color{#3D99F6} 25 \text{ mod }\phi (5)}\right) \text{ (mod 5)} & \small \color{#3D99F6} \text{Euler's totient function }\phi (5) = 4 \\ & = 2\left(3^{\color{#3D99F6} 50 \text{ mod }4} + 8^{\color{#3D99F6} 25 \text{ mod }4}\right) \text{ (mod 5)} \\ & = 2\left(3^{\color{#3D99F6} 2} + 8^{\color{#3D99F6}1}\right) \text{ (mod 5)} \\ & = 2\left(17\right) \text{ (mod 5)} \\ & = \boxed{4} \text{ (mod 5)} \end{aligned}

Thanks so much. I appreciate it!

Đạt Tạ Quang - 4 years, 1 month ago

Log in to reply

Nice problem. I gotta use Vieta's, Newton's, recurrence and Euler's.

Chew-Seong Cheong - 4 years, 1 month ago

What are those blues..?

Vishal Yadav - 4 years, 1 month ago

Log in to reply

In the last part of solution.

Vishal Yadav - 4 years, 1 month ago

Log in to reply

On the right are the explanations.

Chew-Seong Cheong - 4 years, 1 month ago
Ravneet Singh
Apr 17, 2017

On apply vieta's formula, we have x 1 x 2 = 1 x_1x_2 =1 or x 2 = 1 x 1 x_2 = \dfrac{1}{x_1}

Since x 1 , x 2 x_1, x_2 are roots of the given equation so, they must satisfy the equation

Given equation can be written as

x 1 + 1 x 1 = 6 \Longrightarrow x_1 + \dfrac{1}{x_1} = 6

S 1 x 1 + x 2 = 6 S_1\equiv x_1 + x_2 = 6

S 2 x 1 2 + x 2 2 = 34 S_2\equiv x_1^2 + x_2^2 = 34

S 3 x 1 3 + x 2 3 = 198 S_3\equiv x_1^3 + x_2^3 = 198

S 4 x 1 4 + x 2 4 = 1154 S_4\equiv x_1^4 + x_2^4 = 1154

S 5 x 1 5 + x 2 5 = 6726 S_5\equiv x_1^5 + x_2^5 = 6726

we notice the last digit starts repeated after a cycle of 4, therefore in order to find remainder of 50th power when divided by 5,

we only need to see last digit of the 2nd term of the cycle, which is 4 and hence on dividing by 5 we get remainder as 4 \boxed 4 .

Bob Kadylo
Aug 13, 2018

If you are curious: S 50 = 189482250299273866835746159841800035874 S_{50} = 189482250299273866835746159841800035874

When you divide by 5, the remainder is 4 \boxed {4}

0 pending reports

×

Problem Loading...

Note Loading...

Set Loading...